How to prove that adding $n$ to the numerator and denominator will move the resultant fraction close to $1$?












13












$begingroup$


Given a fraction:



$$frac{a}{b}$$



I now add a number $n$ to both numerator and denominator in the following fashion:



$$frac{a+n}{b+n}$$



The basic property is that the second fraction is suppose to closer to 1 than the first one. My question is how can we prove that?



What I have tried:



I know $frac{n}{n} = 1$ so now adding numbers $a$ and $b$ to it would actually "move it away" from $1$. But I cannot understand why $frac{a}{b}$ is actually farther away from 1 than $frac{a+n}{b+n}$.



Why is that? What does it mean to add a number to both the numerator and denominator?










share|cite|improve this question











$endgroup$












  • $begingroup$
    @SikFengCheong I dont understand. What does it mean?
    $endgroup$
    – ng.newbie
    Feb 6 at 8:36






  • 3




    $begingroup$
    If $a lt b$, I just think of the two fractions as batting averages. If you start with a batting average of $a/b$ and get $n$ hits in your next $n$ at bats, your batting average will go up.
    $endgroup$
    – Robert Shore
    Feb 6 at 9:01






  • 9




    $begingroup$
    Adding one to the numerator and denominator of $frac{-2}{-3}$ increases the distance from $1$.
    $endgroup$
    – Martin R
    Feb 6 at 9:17








  • 1




    $begingroup$
    Compute the limit as $n rightarrow infty$.
    $endgroup$
    – chepner
    Feb 6 at 21:00






  • 1




    $begingroup$
    It's not true in general, but it is true for $n, a, b > 0$.
    $endgroup$
    – chepner
    Feb 6 at 21:06
















13












$begingroup$


Given a fraction:



$$frac{a}{b}$$



I now add a number $n$ to both numerator and denominator in the following fashion:



$$frac{a+n}{b+n}$$



The basic property is that the second fraction is suppose to closer to 1 than the first one. My question is how can we prove that?



What I have tried:



I know $frac{n}{n} = 1$ so now adding numbers $a$ and $b$ to it would actually "move it away" from $1$. But I cannot understand why $frac{a}{b}$ is actually farther away from 1 than $frac{a+n}{b+n}$.



Why is that? What does it mean to add a number to both the numerator and denominator?










share|cite|improve this question











$endgroup$












  • $begingroup$
    @SikFengCheong I dont understand. What does it mean?
    $endgroup$
    – ng.newbie
    Feb 6 at 8:36






  • 3




    $begingroup$
    If $a lt b$, I just think of the two fractions as batting averages. If you start with a batting average of $a/b$ and get $n$ hits in your next $n$ at bats, your batting average will go up.
    $endgroup$
    – Robert Shore
    Feb 6 at 9:01






  • 9




    $begingroup$
    Adding one to the numerator and denominator of $frac{-2}{-3}$ increases the distance from $1$.
    $endgroup$
    – Martin R
    Feb 6 at 9:17








  • 1




    $begingroup$
    Compute the limit as $n rightarrow infty$.
    $endgroup$
    – chepner
    Feb 6 at 21:00






  • 1




    $begingroup$
    It's not true in general, but it is true for $n, a, b > 0$.
    $endgroup$
    – chepner
    Feb 6 at 21:06














13












13








13


2



$begingroup$


Given a fraction:



$$frac{a}{b}$$



I now add a number $n$ to both numerator and denominator in the following fashion:



$$frac{a+n}{b+n}$$



The basic property is that the second fraction is suppose to closer to 1 than the first one. My question is how can we prove that?



What I have tried:



I know $frac{n}{n} = 1$ so now adding numbers $a$ and $b$ to it would actually "move it away" from $1$. But I cannot understand why $frac{a}{b}$ is actually farther away from 1 than $frac{a+n}{b+n}$.



Why is that? What does it mean to add a number to both the numerator and denominator?










share|cite|improve this question











$endgroup$




Given a fraction:



$$frac{a}{b}$$



I now add a number $n$ to both numerator and denominator in the following fashion:



$$frac{a+n}{b+n}$$



The basic property is that the second fraction is suppose to closer to 1 than the first one. My question is how can we prove that?



What I have tried:



I know $frac{n}{n} = 1$ so now adding numbers $a$ and $b$ to it would actually "move it away" from $1$. But I cannot understand why $frac{a}{b}$ is actually farther away from 1 than $frac{a+n}{b+n}$.



Why is that? What does it mean to add a number to both the numerator and denominator?







inequality arithmetic fractions






share|cite|improve this question















share|cite|improve this question













share|cite|improve this question




share|cite|improve this question








edited Feb 7 at 10:03









Asaf Karagila

304k32432764




304k32432764










asked Feb 6 at 8:33









ng.newbieng.newbie

302210




302210












  • $begingroup$
    @SikFengCheong I dont understand. What does it mean?
    $endgroup$
    – ng.newbie
    Feb 6 at 8:36






  • 3




    $begingroup$
    If $a lt b$, I just think of the two fractions as batting averages. If you start with a batting average of $a/b$ and get $n$ hits in your next $n$ at bats, your batting average will go up.
    $endgroup$
    – Robert Shore
    Feb 6 at 9:01






  • 9




    $begingroup$
    Adding one to the numerator and denominator of $frac{-2}{-3}$ increases the distance from $1$.
    $endgroup$
    – Martin R
    Feb 6 at 9:17








  • 1




    $begingroup$
    Compute the limit as $n rightarrow infty$.
    $endgroup$
    – chepner
    Feb 6 at 21:00






  • 1




    $begingroup$
    It's not true in general, but it is true for $n, a, b > 0$.
    $endgroup$
    – chepner
    Feb 6 at 21:06


















  • $begingroup$
    @SikFengCheong I dont understand. What does it mean?
    $endgroup$
    – ng.newbie
    Feb 6 at 8:36






  • 3




    $begingroup$
    If $a lt b$, I just think of the two fractions as batting averages. If you start with a batting average of $a/b$ and get $n$ hits in your next $n$ at bats, your batting average will go up.
    $endgroup$
    – Robert Shore
    Feb 6 at 9:01






  • 9




    $begingroup$
    Adding one to the numerator and denominator of $frac{-2}{-3}$ increases the distance from $1$.
    $endgroup$
    – Martin R
    Feb 6 at 9:17








  • 1




    $begingroup$
    Compute the limit as $n rightarrow infty$.
    $endgroup$
    – chepner
    Feb 6 at 21:00






  • 1




    $begingroup$
    It's not true in general, but it is true for $n, a, b > 0$.
    $endgroup$
    – chepner
    Feb 6 at 21:06
















$begingroup$
@SikFengCheong I dont understand. What does it mean?
$endgroup$
– ng.newbie
Feb 6 at 8:36




$begingroup$
@SikFengCheong I dont understand. What does it mean?
$endgroup$
– ng.newbie
Feb 6 at 8:36




3




3




$begingroup$
If $a lt b$, I just think of the two fractions as batting averages. If you start with a batting average of $a/b$ and get $n$ hits in your next $n$ at bats, your batting average will go up.
$endgroup$
– Robert Shore
Feb 6 at 9:01




$begingroup$
If $a lt b$, I just think of the two fractions as batting averages. If you start with a batting average of $a/b$ and get $n$ hits in your next $n$ at bats, your batting average will go up.
$endgroup$
– Robert Shore
Feb 6 at 9:01




9




9




$begingroup$
Adding one to the numerator and denominator of $frac{-2}{-3}$ increases the distance from $1$.
$endgroup$
– Martin R
Feb 6 at 9:17






$begingroup$
Adding one to the numerator and denominator of $frac{-2}{-3}$ increases the distance from $1$.
$endgroup$
– Martin R
Feb 6 at 9:17






1




1




$begingroup$
Compute the limit as $n rightarrow infty$.
$endgroup$
– chepner
Feb 6 at 21:00




$begingroup$
Compute the limit as $n rightarrow infty$.
$endgroup$
– chepner
Feb 6 at 21:00




1




1




$begingroup$
It's not true in general, but it is true for $n, a, b > 0$.
$endgroup$
– chepner
Feb 6 at 21:06




$begingroup$
It's not true in general, but it is true for $n, a, b > 0$.
$endgroup$
– chepner
Feb 6 at 21:06










9 Answers
9






active

oldest

votes


















50












$begingroup$

There's a very simple way to see this. Just take the difference between the two fractions and 1. You want to show that this is smaller in modulus for the second fraction.



You get $$ frac{a}{b} - 1 = frac{a-b}{b} $$ and
$$ frac{a+n}{b+n} -1 = frac{a-b}{b+n} $$



So the second is smaller in modulus (provided $b$ and $n$ are positive, although I supposed it also works if both are negative) because it has same numerator and larger (modulus) denominator, QED.






share|cite|improve this answer











$endgroup$





















    23












    $begingroup$

    Visually: Consider the slope of the line segment from $(0, 0)$ to $(a+n, b+n$):



    enter image description here



    Mathematically (assuming $a, b, n > 0$): The distance
    $$
    left| frac {a+n}{b+n} - 1right| = frac{|a-b|}{b+n}
    $$

    is decreasing in $n$ (and approaches zero for $n to infty$).






    share|cite|improve this answer











    $endgroup$













    • $begingroup$
      [+1] The value of a fraction as a slope is too rarely found. More generally, there are not enough answers that use graphical representations permiting to have a different, complementary, mental representation. It is what I try to do as well in many of my answers.
      $endgroup$
      – Jean Marie
      Feb 7 at 6:50



















    8












    $begingroup$

    You should start by thinking about particular cases. For instance, $dfrac{3+2}{7+2}=dfrac59$, which is indeed closer to $1$ than $dfrac37$.



    Anyway, note that, if $a<b$ (and consequently, $a+n<b+n$, for which $frac ab<1$ and $frac{a+n}{b+n} < 1$), then$$frac{a+n}{b+n}-frac ab=frac{(a+n)b-a(b+n)}{(b+n)b}=frac{n(b-a)}{(b+n)b}>0$$
    This shows $frac{a+n}{b+n}-frac ab>0$, and we already know both are $<1$, so:
    $$frac ab<frac{a+n}{b+n}<1.$$So, yes, $dfrac{a+n}{b+n}$ is closer to $1$ than $dfrac ab$.



    Can you deal with the case $a>b$ now?






    share|cite|improve this answer











    $endgroup$













    • $begingroup$
      I dont understand how $$(a+n)b-a(b+n) = n(b-a)$$
      $endgroup$
      – ng.newbie
      Feb 6 at 8:46












    • $begingroup$
      Because $(a+n)b-a(b+n)=ab+nb-ab-an=n(b-a)$.
      $endgroup$
      – José Carlos Santos
      Feb 6 at 8:47






    • 3




      $begingroup$
      You don't understand that $a+n<b+nifffrac{a+n}{b+n}<1$?
      $endgroup$
      – José Carlos Santos
      Feb 6 at 9:05






    • 2




      $begingroup$
      No! I proved that dividing both sides of the inequality $a+n<b+n$ by $b+n$.
      $endgroup$
      – José Carlos Santos
      Feb 6 at 9:06






    • 1




      $begingroup$
      Yes, that is what I proved, except that I did not mention the “decimal value of that fraction”, whatever that is.
      $endgroup$
      – José Carlos Santos
      Feb 6 at 9:14



















    3












    $begingroup$

    If $b$ and $d$ have the same sign, both
    $$
    frac ab-frac{a+c}{b+d}=frac1bfrac{ad-bc}{b+d}tag1
    $$

    and
    $$
    frac{a+c}{b+d}-frac cd=frac1dfrac{ad-bc}{b+d}tag2
    $$

    also have the same sign. Thus,
    $$
    frac{a+c}{b+d}text{ is between }frac abtext{ and }frac cdtag3
    $$

    Therefore, if $bngt0$,
    $$
    frac{a+n}{b+n}text{ is between }frac abtext{ and }frac nn=1tag4
    $$






    share|cite|improve this answer









    $endgroup$





















      2












      $begingroup$

      Well, $frac{a+n}{b+n} = frac{frac{a}{n}+1}{frac{b}{n}+1}$. So if $nrightarrow infty$, then $frac{a}{n}rightarrow 0$ and $frac{b}{n}rightarrow 0$. Thus $frac{a+n}{b+n}rightarrow 1$.



      As said in the comments, the answer is incorrect in that it does not address precisely what the OP asks, but gives some intuition as to why it is true.






      share|cite|improve this answer











      $endgroup$









      • 1




        $begingroup$
        Comments are not for extended discussion; this conversation has been moved to chat.
        $endgroup$
        – Pedro Tamaroff
        Feb 7 at 14:58






      • 3




        $begingroup$
        The answer is incorrect in that it does not address precisely what the OP asks, but gives some intuition as to why it is true, so please modify it or otherwise delete it, since the vote tally 17/12 which is now at 5 might mislead other users into thinking it is correct. I can also convert it into a comment. Thank you,
        $endgroup$
        – Pedro Tamaroff
        Feb 7 at 14:58








      • 6




        $begingroup$
        @PedroTamaroff: I'm not sure why Xander Henderson's precise objection has been moved to chat, since it is obviously not an extended discussion. In any case, it is false that this answer gives any intuition whatsoever. Surely you won't say that $sqrt[n]{n} > sqrt[n+1]{n+1}$ for positive integer $n$ intuitively 'based' on the fact that $sqrt[n]{n} → 0$ as $n → ∞$? I've just given an explicit example to show that the notion in this post is useless.
        $endgroup$
        – user21820
        Feb 8 at 8:09






      • 4




        $begingroup$
        Contrary to what a mod (!) pretends above, this answer does not give any intuition as to why the fraction (a+n)/(b+n) "is suppose[d] to [be] closer to 1 than" the fraction a/b, which is what the OP is asking. (For the record, I also share @user21820's puzzlement about the reason why a precise mathematical objection formulated in the comments by another user to this effect, has been moved to chat.)
        $endgroup$
        – Did
        Feb 8 at 8:11








      • 1




        $begingroup$
        @user170039 Why these comments? Again, here is the situation: 1. The result is true. 2. Its proof is simple. 3. Such a proof is not in the answer above. 4. The answer above does not "give [any] intuition as to why it is true". 5. A mod deleted some comments explaining why. 6. And asserted wrongly that the answer above "gives some intuition as to why it is true". 7. Some users (but perhaps not all of them) find that mathematically wrong answers should be signalled as such. 8. Some users (but perhaps not all of them) find annoying that a mod endorses some wrong mathematical content ...
        $endgroup$
        – Did
        Feb 8 at 14:12



















      1












      $begingroup$

      Suppose $a,b,n in mathbb Q$, $0 < a < b$ and $n > 0$.



      $$dfrac ab = dfrac{a(b+n)}{b(b+n)} = dfrac{ab+an}{b(b+n)}
      <dfrac{ab+bn}{b(b+n)} = dfrac{b(a+n)}{b(b+n)} = dfrac{a+n}{b+n}
      < dfrac{b+n}{b+n} = 1$$






      share|cite|improve this answer









      $endgroup$





















        0












        $begingroup$

        Let $a=kb$. ($k$ doesnt necessarily have to be an integer). Then:



        $$frac ab = k$$



        $$frac{a+n}{b+n}=frac{k(b+n)-(k-1)n}{b+n}$$
        $$=k-frac{kn-n}{b+n}$$



        Can you show the extra term is positive when $k>1$, and negative when $k<1$? (Hint: let $k=1+t$ for first case and $k=1-t$ for the second)






        share|cite|improve this answer











        $endgroup$













        • $begingroup$
          Can you please how $a+b = k(b+n)-(k-1)n$ ?? I don't understand how you got that.
          $endgroup$
          – ng.newbie
          Feb 6 at 11:21










        • $begingroup$
          Thats not what it is. Its $(a+n)=(bk+n)$, and then we use $n=kn-(k-1)n$
          $endgroup$
          – Rhys Hughes
          Feb 6 at 11:49



















        0












        $begingroup$

        You have to suppose $a,b >0$. Now, it is clear that, if $a<b,;$ i.e. $:smash{dfrac ab}<1$, $a+n<b+n$, hence $smash{dfrac{a+n}{b+n}}<1$, and similarly if $dfrac ab>1$.




        • If $dfrac ab<1$, then $;dfrac ab<dfrac{a+n}{b+n}:(<1)$, which is equivalent to
          $$a(b+n)<b(a+n)iff an<bniff a<b.$$

        • Similar proof that if $dfrac ab>1$, then $;dfrac ab>dfrac{a+n}{b+n}:(>1)$.






        share|cite|improve this answer









        $endgroup$





















          0












          $begingroup$

          Just for the fun of it, since you already received very good answers.



          Perform the long division to get
          $$frac{a+n}{b+n}=1+frac{a-b}nleft(1-frac{b}{n}+frac{b^2}{n^2}-frac{b^3}{n^3} +cdotsright)=1+frac{a-b}nsum_{k=0}^infty (-1)^k left(frac bnright)^k$$






          share|cite|improve this answer









          $endgroup$












            protected by user21820 Feb 7 at 10:54



            Thank you for your interest in this question.
            Because it has attracted low-quality or spam answers that had to be removed, posting an answer now requires 10 reputation on this site (the association bonus does not count).



            Would you like to answer one of these unanswered questions instead?














            9 Answers
            9






            active

            oldest

            votes








            9 Answers
            9






            active

            oldest

            votes









            active

            oldest

            votes






            active

            oldest

            votes









            50












            $begingroup$

            There's a very simple way to see this. Just take the difference between the two fractions and 1. You want to show that this is smaller in modulus for the second fraction.



            You get $$ frac{a}{b} - 1 = frac{a-b}{b} $$ and
            $$ frac{a+n}{b+n} -1 = frac{a-b}{b+n} $$



            So the second is smaller in modulus (provided $b$ and $n$ are positive, although I supposed it also works if both are negative) because it has same numerator and larger (modulus) denominator, QED.






            share|cite|improve this answer











            $endgroup$


















              50












              $begingroup$

              There's a very simple way to see this. Just take the difference between the two fractions and 1. You want to show that this is smaller in modulus for the second fraction.



              You get $$ frac{a}{b} - 1 = frac{a-b}{b} $$ and
              $$ frac{a+n}{b+n} -1 = frac{a-b}{b+n} $$



              So the second is smaller in modulus (provided $b$ and $n$ are positive, although I supposed it also works if both are negative) because it has same numerator and larger (modulus) denominator, QED.






              share|cite|improve this answer











              $endgroup$
















                50












                50








                50





                $begingroup$

                There's a very simple way to see this. Just take the difference between the two fractions and 1. You want to show that this is smaller in modulus for the second fraction.



                You get $$ frac{a}{b} - 1 = frac{a-b}{b} $$ and
                $$ frac{a+n}{b+n} -1 = frac{a-b}{b+n} $$



                So the second is smaller in modulus (provided $b$ and $n$ are positive, although I supposed it also works if both are negative) because it has same numerator and larger (modulus) denominator, QED.






                share|cite|improve this answer











                $endgroup$



                There's a very simple way to see this. Just take the difference between the two fractions and 1. You want to show that this is smaller in modulus for the second fraction.



                You get $$ frac{a}{b} - 1 = frac{a-b}{b} $$ and
                $$ frac{a+n}{b+n} -1 = frac{a-b}{b+n} $$



                So the second is smaller in modulus (provided $b$ and $n$ are positive, although I supposed it also works if both are negative) because it has same numerator and larger (modulus) denominator, QED.







                share|cite|improve this answer














                share|cite|improve this answer



                share|cite|improve this answer








                edited Feb 6 at 14:48

























                answered Feb 6 at 10:27









                tothtoth

                48135




                48135























                    23












                    $begingroup$

                    Visually: Consider the slope of the line segment from $(0, 0)$ to $(a+n, b+n$):



                    enter image description here



                    Mathematically (assuming $a, b, n > 0$): The distance
                    $$
                    left| frac {a+n}{b+n} - 1right| = frac{|a-b|}{b+n}
                    $$

                    is decreasing in $n$ (and approaches zero for $n to infty$).






                    share|cite|improve this answer











                    $endgroup$













                    • $begingroup$
                      [+1] The value of a fraction as a slope is too rarely found. More generally, there are not enough answers that use graphical representations permiting to have a different, complementary, mental representation. It is what I try to do as well in many of my answers.
                      $endgroup$
                      – Jean Marie
                      Feb 7 at 6:50
















                    23












                    $begingroup$

                    Visually: Consider the slope of the line segment from $(0, 0)$ to $(a+n, b+n$):



                    enter image description here



                    Mathematically (assuming $a, b, n > 0$): The distance
                    $$
                    left| frac {a+n}{b+n} - 1right| = frac{|a-b|}{b+n}
                    $$

                    is decreasing in $n$ (and approaches zero for $n to infty$).






                    share|cite|improve this answer











                    $endgroup$













                    • $begingroup$
                      [+1] The value of a fraction as a slope is too rarely found. More generally, there are not enough answers that use graphical representations permiting to have a different, complementary, mental representation. It is what I try to do as well in many of my answers.
                      $endgroup$
                      – Jean Marie
                      Feb 7 at 6:50














                    23












                    23








                    23





                    $begingroup$

                    Visually: Consider the slope of the line segment from $(0, 0)$ to $(a+n, b+n$):



                    enter image description here



                    Mathematically (assuming $a, b, n > 0$): The distance
                    $$
                    left| frac {a+n}{b+n} - 1right| = frac{|a-b|}{b+n}
                    $$

                    is decreasing in $n$ (and approaches zero for $n to infty$).






                    share|cite|improve this answer











                    $endgroup$



                    Visually: Consider the slope of the line segment from $(0, 0)$ to $(a+n, b+n$):



                    enter image description here



                    Mathematically (assuming $a, b, n > 0$): The distance
                    $$
                    left| frac {a+n}{b+n} - 1right| = frac{|a-b|}{b+n}
                    $$

                    is decreasing in $n$ (and approaches zero for $n to infty$).







                    share|cite|improve this answer














                    share|cite|improve this answer



                    share|cite|improve this answer








                    edited Feb 6 at 9:44

























                    answered Feb 6 at 9:39









                    Martin RMartin R

                    28.8k33356




                    28.8k33356












                    • $begingroup$
                      [+1] The value of a fraction as a slope is too rarely found. More generally, there are not enough answers that use graphical representations permiting to have a different, complementary, mental representation. It is what I try to do as well in many of my answers.
                      $endgroup$
                      – Jean Marie
                      Feb 7 at 6:50


















                    • $begingroup$
                      [+1] The value of a fraction as a slope is too rarely found. More generally, there are not enough answers that use graphical representations permiting to have a different, complementary, mental representation. It is what I try to do as well in many of my answers.
                      $endgroup$
                      – Jean Marie
                      Feb 7 at 6:50
















                    $begingroup$
                    [+1] The value of a fraction as a slope is too rarely found. More generally, there are not enough answers that use graphical representations permiting to have a different, complementary, mental representation. It is what I try to do as well in many of my answers.
                    $endgroup$
                    – Jean Marie
                    Feb 7 at 6:50




                    $begingroup$
                    [+1] The value of a fraction as a slope is too rarely found. More generally, there are not enough answers that use graphical representations permiting to have a different, complementary, mental representation. It is what I try to do as well in many of my answers.
                    $endgroup$
                    – Jean Marie
                    Feb 7 at 6:50











                    8












                    $begingroup$

                    You should start by thinking about particular cases. For instance, $dfrac{3+2}{7+2}=dfrac59$, which is indeed closer to $1$ than $dfrac37$.



                    Anyway, note that, if $a<b$ (and consequently, $a+n<b+n$, for which $frac ab<1$ and $frac{a+n}{b+n} < 1$), then$$frac{a+n}{b+n}-frac ab=frac{(a+n)b-a(b+n)}{(b+n)b}=frac{n(b-a)}{(b+n)b}>0$$
                    This shows $frac{a+n}{b+n}-frac ab>0$, and we already know both are $<1$, so:
                    $$frac ab<frac{a+n}{b+n}<1.$$So, yes, $dfrac{a+n}{b+n}$ is closer to $1$ than $dfrac ab$.



                    Can you deal with the case $a>b$ now?






                    share|cite|improve this answer











                    $endgroup$













                    • $begingroup$
                      I dont understand how $$(a+n)b-a(b+n) = n(b-a)$$
                      $endgroup$
                      – ng.newbie
                      Feb 6 at 8:46












                    • $begingroup$
                      Because $(a+n)b-a(b+n)=ab+nb-ab-an=n(b-a)$.
                      $endgroup$
                      – José Carlos Santos
                      Feb 6 at 8:47






                    • 3




                      $begingroup$
                      You don't understand that $a+n<b+nifffrac{a+n}{b+n}<1$?
                      $endgroup$
                      – José Carlos Santos
                      Feb 6 at 9:05






                    • 2




                      $begingroup$
                      No! I proved that dividing both sides of the inequality $a+n<b+n$ by $b+n$.
                      $endgroup$
                      – José Carlos Santos
                      Feb 6 at 9:06






                    • 1




                      $begingroup$
                      Yes, that is what I proved, except that I did not mention the “decimal value of that fraction”, whatever that is.
                      $endgroup$
                      – José Carlos Santos
                      Feb 6 at 9:14
















                    8












                    $begingroup$

                    You should start by thinking about particular cases. For instance, $dfrac{3+2}{7+2}=dfrac59$, which is indeed closer to $1$ than $dfrac37$.



                    Anyway, note that, if $a<b$ (and consequently, $a+n<b+n$, for which $frac ab<1$ and $frac{a+n}{b+n} < 1$), then$$frac{a+n}{b+n}-frac ab=frac{(a+n)b-a(b+n)}{(b+n)b}=frac{n(b-a)}{(b+n)b}>0$$
                    This shows $frac{a+n}{b+n}-frac ab>0$, and we already know both are $<1$, so:
                    $$frac ab<frac{a+n}{b+n}<1.$$So, yes, $dfrac{a+n}{b+n}$ is closer to $1$ than $dfrac ab$.



                    Can you deal with the case $a>b$ now?






                    share|cite|improve this answer











                    $endgroup$













                    • $begingroup$
                      I dont understand how $$(a+n)b-a(b+n) = n(b-a)$$
                      $endgroup$
                      – ng.newbie
                      Feb 6 at 8:46












                    • $begingroup$
                      Because $(a+n)b-a(b+n)=ab+nb-ab-an=n(b-a)$.
                      $endgroup$
                      – José Carlos Santos
                      Feb 6 at 8:47






                    • 3




                      $begingroup$
                      You don't understand that $a+n<b+nifffrac{a+n}{b+n}<1$?
                      $endgroup$
                      – José Carlos Santos
                      Feb 6 at 9:05






                    • 2




                      $begingroup$
                      No! I proved that dividing both sides of the inequality $a+n<b+n$ by $b+n$.
                      $endgroup$
                      – José Carlos Santos
                      Feb 6 at 9:06






                    • 1




                      $begingroup$
                      Yes, that is what I proved, except that I did not mention the “decimal value of that fraction”, whatever that is.
                      $endgroup$
                      – José Carlos Santos
                      Feb 6 at 9:14














                    8












                    8








                    8





                    $begingroup$

                    You should start by thinking about particular cases. For instance, $dfrac{3+2}{7+2}=dfrac59$, which is indeed closer to $1$ than $dfrac37$.



                    Anyway, note that, if $a<b$ (and consequently, $a+n<b+n$, for which $frac ab<1$ and $frac{a+n}{b+n} < 1$), then$$frac{a+n}{b+n}-frac ab=frac{(a+n)b-a(b+n)}{(b+n)b}=frac{n(b-a)}{(b+n)b}>0$$
                    This shows $frac{a+n}{b+n}-frac ab>0$, and we already know both are $<1$, so:
                    $$frac ab<frac{a+n}{b+n}<1.$$So, yes, $dfrac{a+n}{b+n}$ is closer to $1$ than $dfrac ab$.



                    Can you deal with the case $a>b$ now?






                    share|cite|improve this answer











                    $endgroup$



                    You should start by thinking about particular cases. For instance, $dfrac{3+2}{7+2}=dfrac59$, which is indeed closer to $1$ than $dfrac37$.



                    Anyway, note that, if $a<b$ (and consequently, $a+n<b+n$, for which $frac ab<1$ and $frac{a+n}{b+n} < 1$), then$$frac{a+n}{b+n}-frac ab=frac{(a+n)b-a(b+n)}{(b+n)b}=frac{n(b-a)}{(b+n)b}>0$$
                    This shows $frac{a+n}{b+n}-frac ab>0$, and we already know both are $<1$, so:
                    $$frac ab<frac{a+n}{b+n}<1.$$So, yes, $dfrac{a+n}{b+n}$ is closer to $1$ than $dfrac ab$.



                    Can you deal with the case $a>b$ now?







                    share|cite|improve this answer














                    share|cite|improve this answer



                    share|cite|improve this answer








                    edited Feb 6 at 9:16









                    Rhys Hughes

                    6,6491530




                    6,6491530










                    answered Feb 6 at 8:40









                    José Carlos SantosJosé Carlos Santos

                    161k22127232




                    161k22127232












                    • $begingroup$
                      I dont understand how $$(a+n)b-a(b+n) = n(b-a)$$
                      $endgroup$
                      – ng.newbie
                      Feb 6 at 8:46












                    • $begingroup$
                      Because $(a+n)b-a(b+n)=ab+nb-ab-an=n(b-a)$.
                      $endgroup$
                      – José Carlos Santos
                      Feb 6 at 8:47






                    • 3




                      $begingroup$
                      You don't understand that $a+n<b+nifffrac{a+n}{b+n}<1$?
                      $endgroup$
                      – José Carlos Santos
                      Feb 6 at 9:05






                    • 2




                      $begingroup$
                      No! I proved that dividing both sides of the inequality $a+n<b+n$ by $b+n$.
                      $endgroup$
                      – José Carlos Santos
                      Feb 6 at 9:06






                    • 1




                      $begingroup$
                      Yes, that is what I proved, except that I did not mention the “decimal value of that fraction”, whatever that is.
                      $endgroup$
                      – José Carlos Santos
                      Feb 6 at 9:14


















                    • $begingroup$
                      I dont understand how $$(a+n)b-a(b+n) = n(b-a)$$
                      $endgroup$
                      – ng.newbie
                      Feb 6 at 8:46












                    • $begingroup$
                      Because $(a+n)b-a(b+n)=ab+nb-ab-an=n(b-a)$.
                      $endgroup$
                      – José Carlos Santos
                      Feb 6 at 8:47






                    • 3




                      $begingroup$
                      You don't understand that $a+n<b+nifffrac{a+n}{b+n}<1$?
                      $endgroup$
                      – José Carlos Santos
                      Feb 6 at 9:05






                    • 2




                      $begingroup$
                      No! I proved that dividing both sides of the inequality $a+n<b+n$ by $b+n$.
                      $endgroup$
                      – José Carlos Santos
                      Feb 6 at 9:06






                    • 1




                      $begingroup$
                      Yes, that is what I proved, except that I did not mention the “decimal value of that fraction”, whatever that is.
                      $endgroup$
                      – José Carlos Santos
                      Feb 6 at 9:14
















                    $begingroup$
                    I dont understand how $$(a+n)b-a(b+n) = n(b-a)$$
                    $endgroup$
                    – ng.newbie
                    Feb 6 at 8:46






                    $begingroup$
                    I dont understand how $$(a+n)b-a(b+n) = n(b-a)$$
                    $endgroup$
                    – ng.newbie
                    Feb 6 at 8:46














                    $begingroup$
                    Because $(a+n)b-a(b+n)=ab+nb-ab-an=n(b-a)$.
                    $endgroup$
                    – José Carlos Santos
                    Feb 6 at 8:47




                    $begingroup$
                    Because $(a+n)b-a(b+n)=ab+nb-ab-an=n(b-a)$.
                    $endgroup$
                    – José Carlos Santos
                    Feb 6 at 8:47




                    3




                    3




                    $begingroup$
                    You don't understand that $a+n<b+nifffrac{a+n}{b+n}<1$?
                    $endgroup$
                    – José Carlos Santos
                    Feb 6 at 9:05




                    $begingroup$
                    You don't understand that $a+n<b+nifffrac{a+n}{b+n}<1$?
                    $endgroup$
                    – José Carlos Santos
                    Feb 6 at 9:05




                    2




                    2




                    $begingroup$
                    No! I proved that dividing both sides of the inequality $a+n<b+n$ by $b+n$.
                    $endgroup$
                    – José Carlos Santos
                    Feb 6 at 9:06




                    $begingroup$
                    No! I proved that dividing both sides of the inequality $a+n<b+n$ by $b+n$.
                    $endgroup$
                    – José Carlos Santos
                    Feb 6 at 9:06




                    1




                    1




                    $begingroup$
                    Yes, that is what I proved, except that I did not mention the “decimal value of that fraction”, whatever that is.
                    $endgroup$
                    – José Carlos Santos
                    Feb 6 at 9:14




                    $begingroup$
                    Yes, that is what I proved, except that I did not mention the “decimal value of that fraction”, whatever that is.
                    $endgroup$
                    – José Carlos Santos
                    Feb 6 at 9:14











                    3












                    $begingroup$

                    If $b$ and $d$ have the same sign, both
                    $$
                    frac ab-frac{a+c}{b+d}=frac1bfrac{ad-bc}{b+d}tag1
                    $$

                    and
                    $$
                    frac{a+c}{b+d}-frac cd=frac1dfrac{ad-bc}{b+d}tag2
                    $$

                    also have the same sign. Thus,
                    $$
                    frac{a+c}{b+d}text{ is between }frac abtext{ and }frac cdtag3
                    $$

                    Therefore, if $bngt0$,
                    $$
                    frac{a+n}{b+n}text{ is between }frac abtext{ and }frac nn=1tag4
                    $$






                    share|cite|improve this answer









                    $endgroup$


















                      3












                      $begingroup$

                      If $b$ and $d$ have the same sign, both
                      $$
                      frac ab-frac{a+c}{b+d}=frac1bfrac{ad-bc}{b+d}tag1
                      $$

                      and
                      $$
                      frac{a+c}{b+d}-frac cd=frac1dfrac{ad-bc}{b+d}tag2
                      $$

                      also have the same sign. Thus,
                      $$
                      frac{a+c}{b+d}text{ is between }frac abtext{ and }frac cdtag3
                      $$

                      Therefore, if $bngt0$,
                      $$
                      frac{a+n}{b+n}text{ is between }frac abtext{ and }frac nn=1tag4
                      $$






                      share|cite|improve this answer









                      $endgroup$
















                        3












                        3








                        3





                        $begingroup$

                        If $b$ and $d$ have the same sign, both
                        $$
                        frac ab-frac{a+c}{b+d}=frac1bfrac{ad-bc}{b+d}tag1
                        $$

                        and
                        $$
                        frac{a+c}{b+d}-frac cd=frac1dfrac{ad-bc}{b+d}tag2
                        $$

                        also have the same sign. Thus,
                        $$
                        frac{a+c}{b+d}text{ is between }frac abtext{ and }frac cdtag3
                        $$

                        Therefore, if $bngt0$,
                        $$
                        frac{a+n}{b+n}text{ is between }frac abtext{ and }frac nn=1tag4
                        $$






                        share|cite|improve this answer









                        $endgroup$



                        If $b$ and $d$ have the same sign, both
                        $$
                        frac ab-frac{a+c}{b+d}=frac1bfrac{ad-bc}{b+d}tag1
                        $$

                        and
                        $$
                        frac{a+c}{b+d}-frac cd=frac1dfrac{ad-bc}{b+d}tag2
                        $$

                        also have the same sign. Thus,
                        $$
                        frac{a+c}{b+d}text{ is between }frac abtext{ and }frac cdtag3
                        $$

                        Therefore, if $bngt0$,
                        $$
                        frac{a+n}{b+n}text{ is between }frac abtext{ and }frac nn=1tag4
                        $$







                        share|cite|improve this answer












                        share|cite|improve this answer



                        share|cite|improve this answer










                        answered Feb 6 at 13:16









                        robjohnrobjohn

                        268k27308632




                        268k27308632























                            2












                            $begingroup$

                            Well, $frac{a+n}{b+n} = frac{frac{a}{n}+1}{frac{b}{n}+1}$. So if $nrightarrow infty$, then $frac{a}{n}rightarrow 0$ and $frac{b}{n}rightarrow 0$. Thus $frac{a+n}{b+n}rightarrow 1$.



                            As said in the comments, the answer is incorrect in that it does not address precisely what the OP asks, but gives some intuition as to why it is true.






                            share|cite|improve this answer











                            $endgroup$









                            • 1




                              $begingroup$
                              Comments are not for extended discussion; this conversation has been moved to chat.
                              $endgroup$
                              – Pedro Tamaroff
                              Feb 7 at 14:58






                            • 3




                              $begingroup$
                              The answer is incorrect in that it does not address precisely what the OP asks, but gives some intuition as to why it is true, so please modify it or otherwise delete it, since the vote tally 17/12 which is now at 5 might mislead other users into thinking it is correct. I can also convert it into a comment. Thank you,
                              $endgroup$
                              – Pedro Tamaroff
                              Feb 7 at 14:58








                            • 6




                              $begingroup$
                              @PedroTamaroff: I'm not sure why Xander Henderson's precise objection has been moved to chat, since it is obviously not an extended discussion. In any case, it is false that this answer gives any intuition whatsoever. Surely you won't say that $sqrt[n]{n} > sqrt[n+1]{n+1}$ for positive integer $n$ intuitively 'based' on the fact that $sqrt[n]{n} → 0$ as $n → ∞$? I've just given an explicit example to show that the notion in this post is useless.
                              $endgroup$
                              – user21820
                              Feb 8 at 8:09






                            • 4




                              $begingroup$
                              Contrary to what a mod (!) pretends above, this answer does not give any intuition as to why the fraction (a+n)/(b+n) "is suppose[d] to [be] closer to 1 than" the fraction a/b, which is what the OP is asking. (For the record, I also share @user21820's puzzlement about the reason why a precise mathematical objection formulated in the comments by another user to this effect, has been moved to chat.)
                              $endgroup$
                              – Did
                              Feb 8 at 8:11








                            • 1




                              $begingroup$
                              @user170039 Why these comments? Again, here is the situation: 1. The result is true. 2. Its proof is simple. 3. Such a proof is not in the answer above. 4. The answer above does not "give [any] intuition as to why it is true". 5. A mod deleted some comments explaining why. 6. And asserted wrongly that the answer above "gives some intuition as to why it is true". 7. Some users (but perhaps not all of them) find that mathematically wrong answers should be signalled as such. 8. Some users (but perhaps not all of them) find annoying that a mod endorses some wrong mathematical content ...
                              $endgroup$
                              – Did
                              Feb 8 at 14:12
















                            2












                            $begingroup$

                            Well, $frac{a+n}{b+n} = frac{frac{a}{n}+1}{frac{b}{n}+1}$. So if $nrightarrow infty$, then $frac{a}{n}rightarrow 0$ and $frac{b}{n}rightarrow 0$. Thus $frac{a+n}{b+n}rightarrow 1$.



                            As said in the comments, the answer is incorrect in that it does not address precisely what the OP asks, but gives some intuition as to why it is true.






                            share|cite|improve this answer











                            $endgroup$









                            • 1




                              $begingroup$
                              Comments are not for extended discussion; this conversation has been moved to chat.
                              $endgroup$
                              – Pedro Tamaroff
                              Feb 7 at 14:58






                            • 3




                              $begingroup$
                              The answer is incorrect in that it does not address precisely what the OP asks, but gives some intuition as to why it is true, so please modify it or otherwise delete it, since the vote tally 17/12 which is now at 5 might mislead other users into thinking it is correct. I can also convert it into a comment. Thank you,
                              $endgroup$
                              – Pedro Tamaroff
                              Feb 7 at 14:58








                            • 6




                              $begingroup$
                              @PedroTamaroff: I'm not sure why Xander Henderson's precise objection has been moved to chat, since it is obviously not an extended discussion. In any case, it is false that this answer gives any intuition whatsoever. Surely you won't say that $sqrt[n]{n} > sqrt[n+1]{n+1}$ for positive integer $n$ intuitively 'based' on the fact that $sqrt[n]{n} → 0$ as $n → ∞$? I've just given an explicit example to show that the notion in this post is useless.
                              $endgroup$
                              – user21820
                              Feb 8 at 8:09






                            • 4




                              $begingroup$
                              Contrary to what a mod (!) pretends above, this answer does not give any intuition as to why the fraction (a+n)/(b+n) "is suppose[d] to [be] closer to 1 than" the fraction a/b, which is what the OP is asking. (For the record, I also share @user21820's puzzlement about the reason why a precise mathematical objection formulated in the comments by another user to this effect, has been moved to chat.)
                              $endgroup$
                              – Did
                              Feb 8 at 8:11








                            • 1




                              $begingroup$
                              @user170039 Why these comments? Again, here is the situation: 1. The result is true. 2. Its proof is simple. 3. Such a proof is not in the answer above. 4. The answer above does not "give [any] intuition as to why it is true". 5. A mod deleted some comments explaining why. 6. And asserted wrongly that the answer above "gives some intuition as to why it is true". 7. Some users (but perhaps not all of them) find that mathematically wrong answers should be signalled as such. 8. Some users (but perhaps not all of them) find annoying that a mod endorses some wrong mathematical content ...
                              $endgroup$
                              – Did
                              Feb 8 at 14:12














                            2












                            2








                            2





                            $begingroup$

                            Well, $frac{a+n}{b+n} = frac{frac{a}{n}+1}{frac{b}{n}+1}$. So if $nrightarrow infty$, then $frac{a}{n}rightarrow 0$ and $frac{b}{n}rightarrow 0$. Thus $frac{a+n}{b+n}rightarrow 1$.



                            As said in the comments, the answer is incorrect in that it does not address precisely what the OP asks, but gives some intuition as to why it is true.






                            share|cite|improve this answer











                            $endgroup$



                            Well, $frac{a+n}{b+n} = frac{frac{a}{n}+1}{frac{b}{n}+1}$. So if $nrightarrow infty$, then $frac{a}{n}rightarrow 0$ and $frac{b}{n}rightarrow 0$. Thus $frac{a+n}{b+n}rightarrow 1$.



                            As said in the comments, the answer is incorrect in that it does not address precisely what the OP asks, but gives some intuition as to why it is true.







                            share|cite|improve this answer














                            share|cite|improve this answer



                            share|cite|improve this answer








                            edited Feb 7 at 15:04

























                            answered Feb 6 at 8:39









                            WuestenfuxWuestenfux

                            4,5991413




                            4,5991413








                            • 1




                              $begingroup$
                              Comments are not for extended discussion; this conversation has been moved to chat.
                              $endgroup$
                              – Pedro Tamaroff
                              Feb 7 at 14:58






                            • 3




                              $begingroup$
                              The answer is incorrect in that it does not address precisely what the OP asks, but gives some intuition as to why it is true, so please modify it or otherwise delete it, since the vote tally 17/12 which is now at 5 might mislead other users into thinking it is correct. I can also convert it into a comment. Thank you,
                              $endgroup$
                              – Pedro Tamaroff
                              Feb 7 at 14:58








                            • 6




                              $begingroup$
                              @PedroTamaroff: I'm not sure why Xander Henderson's precise objection has been moved to chat, since it is obviously not an extended discussion. In any case, it is false that this answer gives any intuition whatsoever. Surely you won't say that $sqrt[n]{n} > sqrt[n+1]{n+1}$ for positive integer $n$ intuitively 'based' on the fact that $sqrt[n]{n} → 0$ as $n → ∞$? I've just given an explicit example to show that the notion in this post is useless.
                              $endgroup$
                              – user21820
                              Feb 8 at 8:09






                            • 4




                              $begingroup$
                              Contrary to what a mod (!) pretends above, this answer does not give any intuition as to why the fraction (a+n)/(b+n) "is suppose[d] to [be] closer to 1 than" the fraction a/b, which is what the OP is asking. (For the record, I also share @user21820's puzzlement about the reason why a precise mathematical objection formulated in the comments by another user to this effect, has been moved to chat.)
                              $endgroup$
                              – Did
                              Feb 8 at 8:11








                            • 1




                              $begingroup$
                              @user170039 Why these comments? Again, here is the situation: 1. The result is true. 2. Its proof is simple. 3. Such a proof is not in the answer above. 4. The answer above does not "give [any] intuition as to why it is true". 5. A mod deleted some comments explaining why. 6. And asserted wrongly that the answer above "gives some intuition as to why it is true". 7. Some users (but perhaps not all of them) find that mathematically wrong answers should be signalled as such. 8. Some users (but perhaps not all of them) find annoying that a mod endorses some wrong mathematical content ...
                              $endgroup$
                              – Did
                              Feb 8 at 14:12














                            • 1




                              $begingroup$
                              Comments are not for extended discussion; this conversation has been moved to chat.
                              $endgroup$
                              – Pedro Tamaroff
                              Feb 7 at 14:58






                            • 3




                              $begingroup$
                              The answer is incorrect in that it does not address precisely what the OP asks, but gives some intuition as to why it is true, so please modify it or otherwise delete it, since the vote tally 17/12 which is now at 5 might mislead other users into thinking it is correct. I can also convert it into a comment. Thank you,
                              $endgroup$
                              – Pedro Tamaroff
                              Feb 7 at 14:58








                            • 6




                              $begingroup$
                              @PedroTamaroff: I'm not sure why Xander Henderson's precise objection has been moved to chat, since it is obviously not an extended discussion. In any case, it is false that this answer gives any intuition whatsoever. Surely you won't say that $sqrt[n]{n} > sqrt[n+1]{n+1}$ for positive integer $n$ intuitively 'based' on the fact that $sqrt[n]{n} → 0$ as $n → ∞$? I've just given an explicit example to show that the notion in this post is useless.
                              $endgroup$
                              – user21820
                              Feb 8 at 8:09






                            • 4




                              $begingroup$
                              Contrary to what a mod (!) pretends above, this answer does not give any intuition as to why the fraction (a+n)/(b+n) "is suppose[d] to [be] closer to 1 than" the fraction a/b, which is what the OP is asking. (For the record, I also share @user21820's puzzlement about the reason why a precise mathematical objection formulated in the comments by another user to this effect, has been moved to chat.)
                              $endgroup$
                              – Did
                              Feb 8 at 8:11








                            • 1




                              $begingroup$
                              @user170039 Why these comments? Again, here is the situation: 1. The result is true. 2. Its proof is simple. 3. Such a proof is not in the answer above. 4. The answer above does not "give [any] intuition as to why it is true". 5. A mod deleted some comments explaining why. 6. And asserted wrongly that the answer above "gives some intuition as to why it is true". 7. Some users (but perhaps not all of them) find that mathematically wrong answers should be signalled as such. 8. Some users (but perhaps not all of them) find annoying that a mod endorses some wrong mathematical content ...
                              $endgroup$
                              – Did
                              Feb 8 at 14:12








                            1




                            1




                            $begingroup$
                            Comments are not for extended discussion; this conversation has been moved to chat.
                            $endgroup$
                            – Pedro Tamaroff
                            Feb 7 at 14:58




                            $begingroup$
                            Comments are not for extended discussion; this conversation has been moved to chat.
                            $endgroup$
                            – Pedro Tamaroff
                            Feb 7 at 14:58




                            3




                            3




                            $begingroup$
                            The answer is incorrect in that it does not address precisely what the OP asks, but gives some intuition as to why it is true, so please modify it or otherwise delete it, since the vote tally 17/12 which is now at 5 might mislead other users into thinking it is correct. I can also convert it into a comment. Thank you,
                            $endgroup$
                            – Pedro Tamaroff
                            Feb 7 at 14:58






                            $begingroup$
                            The answer is incorrect in that it does not address precisely what the OP asks, but gives some intuition as to why it is true, so please modify it or otherwise delete it, since the vote tally 17/12 which is now at 5 might mislead other users into thinking it is correct. I can also convert it into a comment. Thank you,
                            $endgroup$
                            – Pedro Tamaroff
                            Feb 7 at 14:58






                            6




                            6




                            $begingroup$
                            @PedroTamaroff: I'm not sure why Xander Henderson's precise objection has been moved to chat, since it is obviously not an extended discussion. In any case, it is false that this answer gives any intuition whatsoever. Surely you won't say that $sqrt[n]{n} > sqrt[n+1]{n+1}$ for positive integer $n$ intuitively 'based' on the fact that $sqrt[n]{n} → 0$ as $n → ∞$? I've just given an explicit example to show that the notion in this post is useless.
                            $endgroup$
                            – user21820
                            Feb 8 at 8:09




                            $begingroup$
                            @PedroTamaroff: I'm not sure why Xander Henderson's precise objection has been moved to chat, since it is obviously not an extended discussion. In any case, it is false that this answer gives any intuition whatsoever. Surely you won't say that $sqrt[n]{n} > sqrt[n+1]{n+1}$ for positive integer $n$ intuitively 'based' on the fact that $sqrt[n]{n} → 0$ as $n → ∞$? I've just given an explicit example to show that the notion in this post is useless.
                            $endgroup$
                            – user21820
                            Feb 8 at 8:09




                            4




                            4




                            $begingroup$
                            Contrary to what a mod (!) pretends above, this answer does not give any intuition as to why the fraction (a+n)/(b+n) "is suppose[d] to [be] closer to 1 than" the fraction a/b, which is what the OP is asking. (For the record, I also share @user21820's puzzlement about the reason why a precise mathematical objection formulated in the comments by another user to this effect, has been moved to chat.)
                            $endgroup$
                            – Did
                            Feb 8 at 8:11






                            $begingroup$
                            Contrary to what a mod (!) pretends above, this answer does not give any intuition as to why the fraction (a+n)/(b+n) "is suppose[d] to [be] closer to 1 than" the fraction a/b, which is what the OP is asking. (For the record, I also share @user21820's puzzlement about the reason why a precise mathematical objection formulated in the comments by another user to this effect, has been moved to chat.)
                            $endgroup$
                            – Did
                            Feb 8 at 8:11






                            1




                            1




                            $begingroup$
                            @user170039 Why these comments? Again, here is the situation: 1. The result is true. 2. Its proof is simple. 3. Such a proof is not in the answer above. 4. The answer above does not "give [any] intuition as to why it is true". 5. A mod deleted some comments explaining why. 6. And asserted wrongly that the answer above "gives some intuition as to why it is true". 7. Some users (but perhaps not all of them) find that mathematically wrong answers should be signalled as such. 8. Some users (but perhaps not all of them) find annoying that a mod endorses some wrong mathematical content ...
                            $endgroup$
                            – Did
                            Feb 8 at 14:12




                            $begingroup$
                            @user170039 Why these comments? Again, here is the situation: 1. The result is true. 2. Its proof is simple. 3. Such a proof is not in the answer above. 4. The answer above does not "give [any] intuition as to why it is true". 5. A mod deleted some comments explaining why. 6. And asserted wrongly that the answer above "gives some intuition as to why it is true". 7. Some users (but perhaps not all of them) find that mathematically wrong answers should be signalled as such. 8. Some users (but perhaps not all of them) find annoying that a mod endorses some wrong mathematical content ...
                            $endgroup$
                            – Did
                            Feb 8 at 14:12











                            1












                            $begingroup$

                            Suppose $a,b,n in mathbb Q$, $0 < a < b$ and $n > 0$.



                            $$dfrac ab = dfrac{a(b+n)}{b(b+n)} = dfrac{ab+an}{b(b+n)}
                            <dfrac{ab+bn}{b(b+n)} = dfrac{b(a+n)}{b(b+n)} = dfrac{a+n}{b+n}
                            < dfrac{b+n}{b+n} = 1$$






                            share|cite|improve this answer









                            $endgroup$


















                              1












                              $begingroup$

                              Suppose $a,b,n in mathbb Q$, $0 < a < b$ and $n > 0$.



                              $$dfrac ab = dfrac{a(b+n)}{b(b+n)} = dfrac{ab+an}{b(b+n)}
                              <dfrac{ab+bn}{b(b+n)} = dfrac{b(a+n)}{b(b+n)} = dfrac{a+n}{b+n}
                              < dfrac{b+n}{b+n} = 1$$






                              share|cite|improve this answer









                              $endgroup$
















                                1












                                1








                                1





                                $begingroup$

                                Suppose $a,b,n in mathbb Q$, $0 < a < b$ and $n > 0$.



                                $$dfrac ab = dfrac{a(b+n)}{b(b+n)} = dfrac{ab+an}{b(b+n)}
                                <dfrac{ab+bn}{b(b+n)} = dfrac{b(a+n)}{b(b+n)} = dfrac{a+n}{b+n}
                                < dfrac{b+n}{b+n} = 1$$






                                share|cite|improve this answer









                                $endgroup$



                                Suppose $a,b,n in mathbb Q$, $0 < a < b$ and $n > 0$.



                                $$dfrac ab = dfrac{a(b+n)}{b(b+n)} = dfrac{ab+an}{b(b+n)}
                                <dfrac{ab+bn}{b(b+n)} = dfrac{b(a+n)}{b(b+n)} = dfrac{a+n}{b+n}
                                < dfrac{b+n}{b+n} = 1$$







                                share|cite|improve this answer












                                share|cite|improve this answer



                                share|cite|improve this answer










                                answered Feb 7 at 15:16









                                steven gregorysteven gregory

                                18.1k32258




                                18.1k32258























                                    0












                                    $begingroup$

                                    Let $a=kb$. ($k$ doesnt necessarily have to be an integer). Then:



                                    $$frac ab = k$$



                                    $$frac{a+n}{b+n}=frac{k(b+n)-(k-1)n}{b+n}$$
                                    $$=k-frac{kn-n}{b+n}$$



                                    Can you show the extra term is positive when $k>1$, and negative when $k<1$? (Hint: let $k=1+t$ for first case and $k=1-t$ for the second)






                                    share|cite|improve this answer











                                    $endgroup$













                                    • $begingroup$
                                      Can you please how $a+b = k(b+n)-(k-1)n$ ?? I don't understand how you got that.
                                      $endgroup$
                                      – ng.newbie
                                      Feb 6 at 11:21










                                    • $begingroup$
                                      Thats not what it is. Its $(a+n)=(bk+n)$, and then we use $n=kn-(k-1)n$
                                      $endgroup$
                                      – Rhys Hughes
                                      Feb 6 at 11:49
















                                    0












                                    $begingroup$

                                    Let $a=kb$. ($k$ doesnt necessarily have to be an integer). Then:



                                    $$frac ab = k$$



                                    $$frac{a+n}{b+n}=frac{k(b+n)-(k-1)n}{b+n}$$
                                    $$=k-frac{kn-n}{b+n}$$



                                    Can you show the extra term is positive when $k>1$, and negative when $k<1$? (Hint: let $k=1+t$ for first case and $k=1-t$ for the second)






                                    share|cite|improve this answer











                                    $endgroup$













                                    • $begingroup$
                                      Can you please how $a+b = k(b+n)-(k-1)n$ ?? I don't understand how you got that.
                                      $endgroup$
                                      – ng.newbie
                                      Feb 6 at 11:21










                                    • $begingroup$
                                      Thats not what it is. Its $(a+n)=(bk+n)$, and then we use $n=kn-(k-1)n$
                                      $endgroup$
                                      – Rhys Hughes
                                      Feb 6 at 11:49














                                    0












                                    0








                                    0





                                    $begingroup$

                                    Let $a=kb$. ($k$ doesnt necessarily have to be an integer). Then:



                                    $$frac ab = k$$



                                    $$frac{a+n}{b+n}=frac{k(b+n)-(k-1)n}{b+n}$$
                                    $$=k-frac{kn-n}{b+n}$$



                                    Can you show the extra term is positive when $k>1$, and negative when $k<1$? (Hint: let $k=1+t$ for first case and $k=1-t$ for the second)






                                    share|cite|improve this answer











                                    $endgroup$



                                    Let $a=kb$. ($k$ doesnt necessarily have to be an integer). Then:



                                    $$frac ab = k$$



                                    $$frac{a+n}{b+n}=frac{k(b+n)-(k-1)n}{b+n}$$
                                    $$=k-frac{kn-n}{b+n}$$



                                    Can you show the extra term is positive when $k>1$, and negative when $k<1$? (Hint: let $k=1+t$ for first case and $k=1-t$ for the second)







                                    share|cite|improve this answer














                                    share|cite|improve this answer



                                    share|cite|improve this answer








                                    edited Feb 6 at 8:50

























                                    answered Feb 6 at 8:42









                                    Rhys HughesRhys Hughes

                                    6,6491530




                                    6,6491530












                                    • $begingroup$
                                      Can you please how $a+b = k(b+n)-(k-1)n$ ?? I don't understand how you got that.
                                      $endgroup$
                                      – ng.newbie
                                      Feb 6 at 11:21










                                    • $begingroup$
                                      Thats not what it is. Its $(a+n)=(bk+n)$, and then we use $n=kn-(k-1)n$
                                      $endgroup$
                                      – Rhys Hughes
                                      Feb 6 at 11:49


















                                    • $begingroup$
                                      Can you please how $a+b = k(b+n)-(k-1)n$ ?? I don't understand how you got that.
                                      $endgroup$
                                      – ng.newbie
                                      Feb 6 at 11:21










                                    • $begingroup$
                                      Thats not what it is. Its $(a+n)=(bk+n)$, and then we use $n=kn-(k-1)n$
                                      $endgroup$
                                      – Rhys Hughes
                                      Feb 6 at 11:49
















                                    $begingroup$
                                    Can you please how $a+b = k(b+n)-(k-1)n$ ?? I don't understand how you got that.
                                    $endgroup$
                                    – ng.newbie
                                    Feb 6 at 11:21




                                    $begingroup$
                                    Can you please how $a+b = k(b+n)-(k-1)n$ ?? I don't understand how you got that.
                                    $endgroup$
                                    – ng.newbie
                                    Feb 6 at 11:21












                                    $begingroup$
                                    Thats not what it is. Its $(a+n)=(bk+n)$, and then we use $n=kn-(k-1)n$
                                    $endgroup$
                                    – Rhys Hughes
                                    Feb 6 at 11:49




                                    $begingroup$
                                    Thats not what it is. Its $(a+n)=(bk+n)$, and then we use $n=kn-(k-1)n$
                                    $endgroup$
                                    – Rhys Hughes
                                    Feb 6 at 11:49











                                    0












                                    $begingroup$

                                    You have to suppose $a,b >0$. Now, it is clear that, if $a<b,;$ i.e. $:smash{dfrac ab}<1$, $a+n<b+n$, hence $smash{dfrac{a+n}{b+n}}<1$, and similarly if $dfrac ab>1$.




                                    • If $dfrac ab<1$, then $;dfrac ab<dfrac{a+n}{b+n}:(<1)$, which is equivalent to
                                      $$a(b+n)<b(a+n)iff an<bniff a<b.$$

                                    • Similar proof that if $dfrac ab>1$, then $;dfrac ab>dfrac{a+n}{b+n}:(>1)$.






                                    share|cite|improve this answer









                                    $endgroup$


















                                      0












                                      $begingroup$

                                      You have to suppose $a,b >0$. Now, it is clear that, if $a<b,;$ i.e. $:smash{dfrac ab}<1$, $a+n<b+n$, hence $smash{dfrac{a+n}{b+n}}<1$, and similarly if $dfrac ab>1$.




                                      • If $dfrac ab<1$, then $;dfrac ab<dfrac{a+n}{b+n}:(<1)$, which is equivalent to
                                        $$a(b+n)<b(a+n)iff an<bniff a<b.$$

                                      • Similar proof that if $dfrac ab>1$, then $;dfrac ab>dfrac{a+n}{b+n}:(>1)$.






                                      share|cite|improve this answer









                                      $endgroup$
















                                        0












                                        0








                                        0





                                        $begingroup$

                                        You have to suppose $a,b >0$. Now, it is clear that, if $a<b,;$ i.e. $:smash{dfrac ab}<1$, $a+n<b+n$, hence $smash{dfrac{a+n}{b+n}}<1$, and similarly if $dfrac ab>1$.




                                        • If $dfrac ab<1$, then $;dfrac ab<dfrac{a+n}{b+n}:(<1)$, which is equivalent to
                                          $$a(b+n)<b(a+n)iff an<bniff a<b.$$

                                        • Similar proof that if $dfrac ab>1$, then $;dfrac ab>dfrac{a+n}{b+n}:(>1)$.






                                        share|cite|improve this answer









                                        $endgroup$



                                        You have to suppose $a,b >0$. Now, it is clear that, if $a<b,;$ i.e. $:smash{dfrac ab}<1$, $a+n<b+n$, hence $smash{dfrac{a+n}{b+n}}<1$, and similarly if $dfrac ab>1$.




                                        • If $dfrac ab<1$, then $;dfrac ab<dfrac{a+n}{b+n}:(<1)$, which is equivalent to
                                          $$a(b+n)<b(a+n)iff an<bniff a<b.$$

                                        • Similar proof that if $dfrac ab>1$, then $;dfrac ab>dfrac{a+n}{b+n}:(>1)$.







                                        share|cite|improve this answer












                                        share|cite|improve this answer



                                        share|cite|improve this answer










                                        answered Feb 6 at 10:31









                                        BernardBernard

                                        121k740116




                                        121k740116























                                            0












                                            $begingroup$

                                            Just for the fun of it, since you already received very good answers.



                                            Perform the long division to get
                                            $$frac{a+n}{b+n}=1+frac{a-b}nleft(1-frac{b}{n}+frac{b^2}{n^2}-frac{b^3}{n^3} +cdotsright)=1+frac{a-b}nsum_{k=0}^infty (-1)^k left(frac bnright)^k$$






                                            share|cite|improve this answer









                                            $endgroup$


















                                              0












                                              $begingroup$

                                              Just for the fun of it, since you already received very good answers.



                                              Perform the long division to get
                                              $$frac{a+n}{b+n}=1+frac{a-b}nleft(1-frac{b}{n}+frac{b^2}{n^2}-frac{b^3}{n^3} +cdotsright)=1+frac{a-b}nsum_{k=0}^infty (-1)^k left(frac bnright)^k$$






                                              share|cite|improve this answer









                                              $endgroup$
















                                                0












                                                0








                                                0





                                                $begingroup$

                                                Just for the fun of it, since you already received very good answers.



                                                Perform the long division to get
                                                $$frac{a+n}{b+n}=1+frac{a-b}nleft(1-frac{b}{n}+frac{b^2}{n^2}-frac{b^3}{n^3} +cdotsright)=1+frac{a-b}nsum_{k=0}^infty (-1)^k left(frac bnright)^k$$






                                                share|cite|improve this answer









                                                $endgroup$



                                                Just for the fun of it, since you already received very good answers.



                                                Perform the long division to get
                                                $$frac{a+n}{b+n}=1+frac{a-b}nleft(1-frac{b}{n}+frac{b^2}{n^2}-frac{b^3}{n^3} +cdotsright)=1+frac{a-b}nsum_{k=0}^infty (-1)^k left(frac bnright)^k$$







                                                share|cite|improve this answer












                                                share|cite|improve this answer



                                                share|cite|improve this answer










                                                answered 21 hours ago









                                                Claude LeiboviciClaude Leibovici

                                                121k1157133




                                                121k1157133

















                                                    protected by user21820 Feb 7 at 10:54



                                                    Thank you for your interest in this question.
                                                    Because it has attracted low-quality or spam answers that had to be removed, posting an answer now requires 10 reputation on this site (the association bonus does not count).



                                                    Would you like to answer one of these unanswered questions instead?



                                                    Popular posts from this blog

                                                    Biblatex bibliography style without URLs when DOI exists (in Overleaf with Zotero bibliography)

                                                    ComboBox Display Member on multiple fields

                                                    Is it possible to collect Nectar points via Trainline?